Uso de la ley de circuito de Ampere para un cable infinitamente largo y un cable de longitud dada

De acuerdo con la Ley de Circuito de Ampere :

ingrese la descripción de la imagen aquí

Ahora considere dos alambres rectos, cada uno transportando corriente I, uno de longitud infinita y otro de longitud finita l. Si necesita averiguar el campo magnético debido a cada uno, en un punto (X) cuya distancia perpendicular al cable es d.

Obtienes un campo magnético como m I 2 π d . Lo mismo para ambos.

Pero,

El campo magnético debido a un alambre infinitamente largo es: m I 2 π d

Campo magnético debido a un alambre de longitud finita l: m I ( pecado ( PAG ) + pecado ( q ) ) 2 π d , donde P y Q son los ángulos subtendidos en el punto por los extremos del cable.

¿Por qué obtenemos un valor incorrecto al usar la ley del circuito de Ampere?

Respuestas (1)

Hay dos cosas a notar aquí.

  1. Solo puedes hacer la tarea B d yo = 2 π d B ( d ) si la situación es radialmente simétrica.
  2. En el caso de un cable finito , tiene acumulación de carga en los extremos o aún no ha especificado toda la distribución de corriente. Pregunta: ¿puede especificar una ruta de retorno radialmente simétrica y, de ser así, espera que compense la diferencia?
Con respecto al caso finito, ¿es posible obtener el campo B justo en el extremo finito de un cable semi-infinito que se extiende hasta el infinito solo en una dirección? Sé que debo tener en cuenta la acumulación de carga en el extremo finito, ya que generará un campo E dependiente del tiempo, por lo tanto, una corriente de desplazamiento. Sin embargo, desconozco los detalles matemáticos, ya que el campo de una carga puntual (suponiendo que se genera una carga puntual en el extremo finito del cable) no está definido en el origen. r = 0 . Lo necesitamos en la integral para obtener la corriente de desplazamiento de la densidad de corriente.
Supongo que en muchos casos se podría suponer que el cable termina en un condensador esférico pequeño pero de tamaño finito, evitando así la necesidad de una singularidad. Luego elija su superficie para que no se cruce con el capacitor.
¿Puede consultar mi última pregunta en este sitio physics.stackexchange.com/questions/182785/…
Recientemente publiqué la misma pregunta pero una ligera variación que muestra simetría al considerar el campo en el centro, es decir, α = β